Course Content
All Previous Years Krok 2 Papers with Explanations
About Lesson


Question From ( 151 To 200 )


151. A 3 year old boy has petechial eruption. Examination revealed no other pathological changes. Thrombocyte number is 20 · 109g/l; haemoglobin and leukocyte concentration is normal. What is the most probable diagnosis?

A. Immune thrombocytopenic purpura

B. Schonlein-Henoch disease

C. Disseminated intravascular coagulopathy

D. Acute lymphoblastic leukemia

E. Systemic lupus erythematosus


Answer:  Immune thrombocytopenic purpura

Explanation

ITP is a disorder in which there is a decreased number of platelets in the blood, leading to bleeding and bruising. The presence of petechial eruptions and a low platelet count (20 · 109g/l) in this 3-year-old boy suggest a diagnosis of ITP.   Schonlein-Henoch disease is a type of vasculitis that can cause petechial eruptions, but it is typically associated with joint pain, abdominal pain, and kidney involvement.

Disseminated intravascular coagulopathy (DIC) is a condition in which there is abnormal clotting and bleeding, and it is typically associated with other underlying conditions such as sepsis, trauma, or cancer. The normal levels of hemoglobin and leukocytes in this case make DIC less likely.

Acute lymphoblastic leukemia (ALL) is a type of cancer that can cause low platelet counts and petechial eruptions, but it is usually associated with other symptoms such as fatigue, fever, and bone pain.

Systemic lupus erythematosus (SLE) is an autoimmune disorder that can cause thrombocytopenia and skin rash, but it is less common in young children and is typically associated with other symptoms such as joint pain, fatigue, and fever.   In summary, the most probable diagnosis in this case is ITP, which is a disorder characterized by a decreased number of platelets and bleeding manifestations such as petechial eruptions.


152. Indices that characterize population health include demographic indices. What environment is used for calculation of these indices?

A. Population number

B. Employment number

C. Number of hospitalized people

D. Number of patients

E. Number of population being liable to preventive examination


Answer: Population number

Explanation

Demographic indices are used to measure various aspects of population health, such as birth rates, death rates, life expectancy, and disease prevalence. These indices are calculated based on data related to the size, structure, and distribution of a population.  

The environment used for calculation of demographic indices is the population itself, which is typically defined as the total number of individuals living in a particular geographic area, such as a country, region, or city. The population number is a critical factor in calculating many demographic indices, as it provides the denominator for calculating rates and other measures of health and disease.  

Employment number, number of hospitalized people, number of patients, and number of population being liable to preventive examination are all important factors related to population health, but they are not used directly in the calculation of demographic indices. These factors may be used to measure specific aspects of population health or to inform public health policy and interventions, but they are not part of the standard set of demographic indices used to assess overall population health.


153. An infant is full-term. Delivery was pathological, with breech presentation. Examination of the infant revealed limited abduction of the right leg to 50o, positive “clicking”symptom on the right, asymmetric inguinal folds. What is the most probable diagnosis?

A. Inborn dislocation of the right hip

B. Inborn dislocation of both hips

C. Varus deformity of both femoral necks

D. Fracture of both femoral necks

E. Right hip dysplasia


Answer: Inborn dislocation of the right hip

 Explanation

Inborn dislocation of the hip (IDH) is a condition in which the ball of the hip joint is not properly seated in the socket. It is more common in females and breech presentation during delivery is a known risk factor.

The signs and symptoms of IDH can include limited range of motion, asymmetric inguinal folds, and a positive “click” sign when the hip is manipulated.   In this case, the infant has limited abduction of the right leg to 50 degrees, a positive “clicking” symptom on the right, and asymmetric inguinal folds, which are all consistent with a diagnosis of IDH.

The fact that the left hip is not affected suggests that the dislocation is unilateral.   Varus deformity of both femoral necks and fracture of both femoral necks are less likely diagnoses in this case, as they would typically present with different signs and symptoms.

Right hip dysplasia is also a possible diagnosis, but it is less likely given the specific signs and symptoms described in the case.   In summary, the most probable diagnosis in this case is inborn dislocation of the right hip, which is a condition in which the ball of the hip joint is not properly seated in the socket and can be associated with limited range of motion, asymmetric inguinal folds, and a positive “click” sign on examination.


154. A 30 year old man complains of intense pain, reddening of skin, edema in the ankle-joint area, fever up to 39oC. There was an acute onset of the illness. In the past there were similar attacks lasting 5-6 days without residual changes in the joint. The skin over the joint is hyperemic and ill-defined, without infiltrative bank on the periphery. What is the most likely diagnosis?

A. Gout

B. Infectious arthritis

C. Rheumatoid arthritis

D. Erysipelatous inflammation

E. Osteoarthritis


Answer:  Gout

 Explanation

Gout is a type of arthritis that occurs when uric acid crystals build up in the joints, leading to intense pain, inflammation, and swelling. The symptoms typically come on suddenly and can be associated with fever and other signs of inflammation. Recurrent attacks of gout are common, and they can be triggered by factors such as diet, alcohol consumption, and certain medications. The most commonly affected joint in gout is the big toe, but other joints, such as the ankle, can also be affected.  

In this case, the patient is experiencing intense pain, reddening of skin, edema in the ankle-joint area, and fever up to 39°C, all of which are consistent with a diagnosis of gout. The fact that there have been similar attacks in the past without residual changes in the joint further supports this diagnosis.

The skin over the joint is hyperemic and ill-defined, without an infiltrative bank on the periphery, which is also typical of gout.   Infectious arthritis, rheumatoid arthritis, erysipelatous inflammation, and osteoarthritis are all possible differential diagnoses, but they are less likely given the specific signs and symptoms described in the case. Infectious arthritis is typically associated with a known infection elsewhere in the body, and the skin over the joint is more likely to show signs of infection, such as redness, warmth, and pus.

Rheumatoid arthritis is a chronic autoimmune disorder that typically affects multiple joints, and it is less likely to present with such an acute onset of symptoms. Erysipelatous inflammation is a bacterial skin infection that can cause similar symptoms, but it typically presents with more distinct borders and is associated with a known portal of entry.

Osteoarthritis is a degenerative joint disease that typically presents with gradual onset of symptoms and is less likely to cause such acute inflammation.   In summary, the most likely diagnosis in this case is gout, which is a type of arthritis that can cause intense pain, inflammation, and swelling in the affected joint, and is often triggered by dietary or lifestyle factors.


155. The results of 5 year monitoring allowed to estimate the level of environmental influence upon health indices of popultaion. What statistic method should be chosen?

A. Calculation of correlation coefficient

B. Calculation of conformity coefficient

C. Calculation of coefficient of difference validity

D. Calculation of regression coefficient

E. Calculation of dynamic indices


Answer: Calculation of correlation coefficient

 Explanation

Correlation coefficient is a statistical measure that indicates the degree of association between two variables. In this case, the environmental factors and health indices of the population can be considered as two variables. By calculating the correlation coefficient, we can determine the strength and direction of the relationship between these two variables.

If there is a strong positive correlation between the environmental factors and health indices, it suggests that the environment has a significant influence on the health of the population.   Conformity coefficient and coefficient of difference validity are not appropriate methods for this type of analysis. Conformity coefficient is used to compare two sets of observations to see how closely they match. Coefficient of difference validity is used to compare the validity of two different measures of the same construct.

These methods are not suitable for estimating the level of environmental influence on health indices.   Calculation of regression coefficient is another statistical method that can be used to analyze the relationship between two variables. However, it is more appropriate when one variable is considered to be the dependent variable and the other variable is considered to be the independent variable.

In this case, it is not clear which variable is dependent and which is independent, so regression analysis may not be the best method to use.   Calculation of dynamic indices is a method used to analyze changes in data over time.

While this method may be useful for monitoring trends in health indices over time, it may not be the most appropriate method for estimating the level of environmental influence on health indices.   In summary, the most appropriate statistical method to estimate the level of environmental influence upon health indices of a population based on 5-year monitoring would be the calculation of correlation coefficient.


156. A 37 year old miner has lifted significant loads and afterwards felt pain in the lumbar area irradiating to his left leg. He walks slowly and carefully. Lumbar lordosis is flattened. There is also left-sided scoliosis and tension of paravertebral muscles. Neri’s and Dejerine’s symptoms are positive, there is Lasegue’s sign on the left from the angle of 350. What method will help to specify the diagnosis?

A. CT of lumbosacral part of vertebral column

B. Lumbal puncture

C. Renal sonography

D. Descending myelography

E. Electromyography


Answer: CT of lumbosacral part of vertebral column

 Explanation

The patient’s symptoms, including pain in the lumbar area irradiating to the left leg, flattened lumbar lordosis, left-sided scoliosis, and tension of paravertebral muscles, suggest a possible diagnosis of lumbar disc herniation. The positive Neri’s and Dejerine’s symptoms, as well as Lasegue’s sign on the left from the angle of 35 degrees, are also consistent with this diagnosis.  

CT of the lumbosacral part of the vertebral column is a diagnostic imaging technique that can help visualize the spinal structures and identify any abnormalities, such as herniated discs or spinal stenosis, which may be contributing to the patient’s symptoms. CT is particularly useful in identifying bony abnormalities and can provide detailed images of the spinal structures, allowing for more accurate diagnosis.  

Lumbar puncture, renal sonography, descending myelography, and electromyography are not appropriate methods for diagnosing lumbar disc herniation. Lumbar puncture is used to obtain cerebrospinal fluid for analysis and is not used to diagnose spinal disorders. Renal sonography is used to evaluate the kidneys and urinary tract and is not relevant to this case. Descending myelography is an invasive imaging procedure that involves injecting a contrast agent into the spinal canal and is used to diagnose spinal cord disorders.

Electromyography is a diagnostic technique that measures the electrical activity of muscles and nerves and is used to diagnose neuromuscular disorders.   In summary, CT of the lumbosacral part of the vertebral column is the most appropriate method to help specify the diagnosis in this case, as it can provide detailed images of the spinal structures and identify any abnormalities that may be contributing to the patient’s symptoms.


157. A 47 year old male patient got a flame burn of trunk and upper extremities and was delivered to the hospital. The patient is in grave condition, confused mental state, with fever. AP- 80/50 mm Hg, Ps- 118 bpm. It was locally stated that the patient got III B degree burns with total area of 20%. What medical actions should be taken?

A. Injection of narcotic analgetics and powdered blood substitutes

B. Primary surgical pocessing

C. Administration of detoxicating blood substitutes

D. Necrotomy of burn surface, haemotransfusion

E. Antibacterial and detoxicating therapy


Answer: Injection of narcotic analgetics and powdered blood substitutes

 Explanation

The patient’s symptoms, including confusion, fever, and tachycardia, suggest a possible systemic inflammatory response syndrome (SIRS), which can occur in response to severe burns. The patient’s low blood pressure also indicates hypovolemia, which can occur due to fluid loss from the burn injury.  

The appropriate medical management for this patient would include the following:  
1. Fluid resuscitation: The patient should receive intravenous fluids to restore blood volume and maintain blood pressure. This can be done using crystalloid solutions, such as normal saline or lactated Ringer’s solution.  

2. Pain management: The patient should receive appropriate pain management, which may include narcotic analgesics, such as morphine, to control pain related to the burn injury.  

3. Antibacterial therapy: The risk of infection is high in patients with burn injuries. Therefore, the patient should receive broad-spectrum antibiotics to prevent and treat any potential infections.  

4. Detoxification: The patient should receive detoxification therapies, such as intravenous administration of N-acetylcysteine or other antioxidants, to prevent or reduce oxidative damage and the risk of acute kidney injury.  

5. Nutritional support: The patient should receive appropriate nutritional support, including enteral or parenteral feeding, to maintain adequate nutrition and prevent malnutrition.  

Necrotomy of the burn surface and blood transfusion would not be appropriate initial interventions in this patient, as they are invasive procedures that carry risks and complications. Primary surgical processing and administration of detoxifying blood substitutes are not specific interventions for burn injuries and may not address the patient’s specific needs.   In summary, the most appropriate medical actions to take in this case would be


158. A 54 year old female patient was admitted to the hospital with evident acrocyanosis, swollen cervical veins, enlarged liver, ascites. Cardiac borders are dilated. Heart sounds cannot be auscultated, apical beat is undetectable. AP is 100/50 mm Hg. X-ray picture of chest shows enlarged heart shadow in form of a trapezium. What pathology might have caused these symptoms?

A. Cardiac tamponade

B. Exudative pleuritis

C. Complex heart defect

D. Acute cardiac insufficiency

E. Hiatal hernia


Answer:  Cardiac tamponade

 Explanation

The symptoms described in this case are most consistent with cardiac tamponade, which is a medical emergency that occurs when excess fluid accumulates in the pericardium, the sac surrounding the heart. The fluid buildup puts pressure on the heart, which can lead to a decrease in cardiac output and ultimately heart failure.  

The patient’s symptoms, including acrocyanosis, swollen cervical veins, enlarged liver, ascites, and dilated cardiac borders, are all consistent with a diagnosis of cardiac tamponade. The absence of heart sounds and undetectable apical beat suggest that the excess fluid is preventing the heart from functioning properly.

The enlarged heart shadow on chest X-ray is also consistent with this diagnosis.   Exudative pleuritis, complex heart defect, acute cardiac insufficiency, and hiatal hernia are all possible differential diagnoses, but they are less likely given the specific signs and symptoms described in the case.

Exudative pleuritis is a condition in which excess fluid accumulates in the pleural space, but this typically presents with symptoms such as shortness of breath, chest pain, and cough. A complex heart defect would be present from birth and is less likely to present in a 54-year-old patient. Acute cardiac insufficiency, or congestive heart failure, can cause similar symptoms, but


159. 15 minutes after the second vaccination with diphteria and tetanus toxoids and pertussis vaccine a 4 month old boy manifested symptoms of Quincke’s edema. What medication should be applied for emergency aid?

A. Prednisolone

B. Heparin

C. Adrenalin

D. Furosemide

E. Seduxen


Answer:  Prednisolone

 Explanation

In this case, the most appropriate medication for emergency aid would be A. Prednisolone.   Quincke’s edema, also known as angioedema, is a type of allergic reaction that can occur in response to a variety of triggers, including vaccines. The symptoms typically include swelling of the face, lips, tongue, and throat, which may cause difficulty breathing or swallowing.  

In this case, the patient has developed Quincke’s edema shortly after receiving a vaccination. The immediate treatment for Quincke’s edema involves administering an antihistamine and a corticosteroid to reduce inflammation and swelling.   Prednisolone is a corticosteroid that is often used to treat allergic reactions, including Quincke’s edema.

It works by reducing inflammation and swelling in the affected tissues. It can be administered orally or intravenously.   Adrenalin (epinephrine) is another medication that can be used to treat allergic reactions. It works by constricting blood vessels and relaxing the airways, which can help to alleviate symptoms such as difficulty breathing.

However, in this case, prednisolone would be the first line of treatment, and epinephrine would only be used if the patient’s symptoms do not respond to prednisolone.   Heparin, furosemide, and seduxen are not appropriate medications for Quincke’s edema and would not be useful in this case.   In summary, the most appropriate medication for emergency aid in this case would be prednisolone, which is a corticosteroid that can help to reduce inflammation and swelling associated with Quincke’s edema.


160. A 16 year old female patient underwent an operation on account of diffuse toxic goiter of the III-IV degree 12 years ago. Now she has recurrence of thyrotoxicosis. The patient was offered operative intervention, but it is necessary first to localize the functioning gland tissue. What method should be applied for this purpose?

A. Gland scanning

B. USI

C. Puncture aspiration biopsy

D. Roentgenography of neck

E. Roentgenography of esophagus


Answer:  Gland scanning

 Explanation

The most appropriate method for localizing functioning gland tissue in a patient with recurrent thyrotoxicosis would be A. Gland scanning, also known as a thyroid scan.   A thyroid scan is a nuclear medicine imaging technique that uses radioactive iodine to visualize the thyroid gland.

The patient is given a small amount of radioactive iodine, which is taken up by the thyroid gland. A special camera is then used to create images of the thyroid gland, which can help to identify areas of functioning gland tissue and determine the extent of disease.   Gland scanning is a non-invasive and relatively safe procedure that is commonly used to diagnose and monitor thyroid disorders, including thyrotoxicosis. It can also help to distinguish between different types of thyroid disease, such as Graves’ disease and toxic nodular goiter.  

USI (ultrasound imaging) is another imaging technique that can be used to evaluate the thyroid gland, but it is less useful for localizing functioning gland tissue. Puncture aspiration biopsy (fine-needle aspiration) is a diagnostic procedure that involves taking a sample of thyroid tissue for analysis, but it is not used for localizing functioning gland tissue.

Roentgenography (X-ray imaging) of the neck and esophagus are not useful for evaluating thyroid function or localizing functioning gland tissue.   In summary, gland scanning is the most appropriate method for localizing functioning gland tissue in a patient with recurrent thyrotoxicosis, as it is a non-invasive and effective imaging technique that can help to identify areas of disease and guide further treatment decisions.


161. Examination of a 26 year old female patient revealed a node in the right lobe of thyroid gland. The node appeared no earlier than 3 months ago. The patient associates this node with stress. She doesn’t complain either about pain or enlargement of the node. Ultrasonic scanning revealed a 2×2,5 cm large node in the inferior part of the right lobe of thyroid gland. What treatment should be administered?

A. Surgical intervention

B. Conservative therapy

C. Dynamic observation

D. No need for treatment

E. –


Answer:  Surgical intervention

 Explanation

The presence of a thyroid nodule, especially one that is rapidly growing or greater than 1 cm in size, raises concern for possible malignancy. While most thyroid nodules are benign, it is important to evaluate the nodule further to rule out the possibility of thyroid cancer.  

Surgical intervention, such as thyroidectomy or lobectomy, is the most definitive treatment for thyroid nodules. Surgery is typically recommended for nodules that are suspicious for malignancy or causing compression of nearby structures, such as the trachea or esophagus.  

In this case, the patient has a 2×2.5 cm large nodule in the inferior part of the right lobe of the thyroid gland, which is a significant size. While the patient does not report pain or enlargement of the node, the presence of the nodule for only 3 months and the lack of a clear history of benignity warrant further evaluation.

Therefore, surgical intervention is the most appropriate treatment in this case.   Conservative therapy, such as levothyroxine therapy, is not typically indicated for thyroid nodules, especially if they are suspected to be malignant. Dynamic observation (also known as watchful waiting) may be appropriate for small nodules that are not causing symptoms and are likely benign, but given the size and recent onset of the nodule in this case, observation alone would not be appropriate.  

In summary, surgical intervention is the most appropriate treatment for a 2×2.5 cm thyroid nodule that is suspicious for malignancy, even if the patient does not report pain or enlargement of the nodule.


162. During examination a patient is unconscious, his skin is dry and hot, face hyperemia is present. The patient has Kussmaul’s respiration, there is also smell of acetone in the air. Symptoms of peritoneum irritation are positive. Blood sugar is at the rate of 33 millimole/l. What emergency actions should be taken?

A. Intravenous infusion of short-acting insulin

B. Intravenous infusion of glucose along with insulin

C. Introduction of long-acting insulin

D. Intravenous infusion of neohaemodesum along with glutamic acid

E. Intravenous infusion of sodium chloride saline


Answer: Intravenous infusion of short-acting insulin

 Explanation

The symptoms described in this case are consistent with diabetic ketoacidosis (DKA), which is a life-threatening complication of uncontrolled diabetes. The most appropriate emergency action to take in this case would be A. Intravenous infusion of short-acting insulin.  

DKA occurs when the body produces high levels of ketones, which are byproducts of the breakdown of fat for energy. This can occur when there is a lack of insulin to help the body use glucose for energy, leading to hyperglycemia (high blood sugar) and metabolic acidosis.

The symptoms of DKA include Kussmaul’s respiration, acetone breath, dehydration, abdominal pain, and altered mental status.   In this case, the patient is unconscious and has a blood sugar level of 33 millimole/l, which is extremely high. The recommended treatment for DKA is rapid-acting insulin, which is typically administered by intravenous infusion.

This helps to lower blood sugar levels and reduce the production of ketones.   Intravenous infusion of glucose alone would not be appropriate in this case, as it would further increase blood sugar levels and worsen the metabolic acidosis.

Long-acting insulin or neohaemodesum with glutamic acid would not be appropriate for emergency treatment of DKA. Intravenous infusion of sodium chloride saline may be used to help correct dehydration, but insulin therapy is the primary treatment for DKA.   In summary, in a case of suspected DKA with high blood sugar levels and metabolic acidosis, the most appropriate emergency action would be to administer intravenous infusion of short-acting insulin.


163. A patient complains about evaginations in the region of anus that appear during defecation and need to be replaced. Examination with anoscope revealed 1×1 cm large evaginations of mucosa above the pectineal line. What is the most probable diagnosis?

A. Internal hemorrhoids

B. Acute paraproctitis

C. External hemorrhoids

D. Anal fissure

E. –


Answer: Internal hemorrhoids

 Explanation

In this case, the most probable diagnosis is A. Internal hemorrhoids.   Internal hemorrhoids are swollen veins that develop inside the rectum. They are located above the pectineal line, which is an anatomical landmark that separates the upper and lower parts of the anal canal. Symptoms of internal hemorrhoids include evaginations or protrusions of tissue during defecation, which may require manual replacement, as well as bleeding, itching, and discomfort.  

The finding of 1×1 cm large evaginations of mucosa above the pectineal line on anoscopy is consistent with internal hemorrhoids. Anoscopy is a diagnostic procedure that involves inserting a small, tubular instrument called an anoscope into the anus to examine the rectum and anal canal.  

Acute paraproctitis is a bacterial infection of the tissues around the anus and rectum, which typically presents with symptoms such as pain, swelling, and fever. External hemorrhoids are swollen veins that develop outside the anus and can cause similar symptoms to internal hemorrhoids, but they are located below the pectineal line. Anal fissure is a tear in the lining of the anal canal, which can cause pain and bleeding during bowel movements.  

In summary, the most probable diagnosis in this case is internal hemorrhoids, which are swollen veins located above the pectineal line that can cause evaginations or protrusions of tissue during defecation, as well as other symptoms such as bleeding and itching.


164. A 34 year old patient was delivered to the hospital because of follicular tonsillitis charactirized by high temperature. The patient has been abusing alcohol for 12 years. In the evening on the day of hospitalization he became anxious, couldn’t stay in bed, left his ward several times and applied to the staff on duty with different complaints. He reported about seeing alot of spiders and flies in his ward as well as abou hearing threats from the corridor. He was exasperated by the fact that other patients didn’t hear them. He lost also spatial orientation. What psychopathological syndrome is it?

A. Delirious

B. Oneiric

C. Amentive

D. Twilight state

E. Asthenic confusion


Answer: Delirious

 Explanation

Delirium is a neuropsychiatric syndrome characterized by acute onset of cognitive impairment, including changes in attention, awareness, and perception. It is commonly seen in elderly patients, patients with underlying medical conditions, and those with a history of alcohol abuse.  

In this case, the patient presented with fever and follicular tonsillitis, which may have contributed to the development of delirium. However, the patient’s history of alcohol abuse is also a significant risk factor for delirium.   The patient’s symptoms, including anxiety, restlessness, hallucinations (seeing spiders and flies), and misinterpretation of the environment (hearing threats that others did not hear), are consistent with delirium. Loss of spatial orientation is also a common feature of delirium.  

Oneiric syndrome is characterized by vivid and complex dreams that the patient may have difficulty distinguishing from reality. Amentive syndrome is characterized by apathy, lack of initiative, and decreased motivation. Twilight state is a term used to describe altered states of consciousness that may occur during hypnosis or seizures.

Asthenic confusion is a type of cognitive impairment seen in patients with certain neurological conditions.   In summary, the most likely psychopathological syndrome in this case is A. Delirious syndrome, which is characterized by acute onset of cognitive impairment, including changes in attention, awareness, and perception, and is commonly seen in patients with underlying medical conditions or a history of alcohol abuse.


165. On the third day of life an infant’s skin got icteric colouring. The child was born with body weight of 3,200 kg, body length of 52 cm. The child is active. There is puerile respiration above the lungs. Respiratory rate is 36/min, heart sounds are rhythmic, heart rate is 130/min. Abdomen is soft, liver comes out from the edge of costal arch by 2 cm, spleen is not palpable. Feces are in form of meconium. What is the most probable diagnosis?

A. Physiologic jaundice

B. Hemolytic disease of newborn

C. Neonatal sepsis

D. Minkowsky-Shauffard disease

E. Biliary tracts atresia


Answer: Physiologic jaundice

 Explanation

Physiologic jaundice is a common condition in newborns, typically occurring in the first week of life. It is caused by an increase in bilirubin levels due to the breakdown of fetal red blood cells and delayed maturation of the liver’s ability to clear bilirubin.

Physiologic jaundice usually resolves within 1-2 weeks without treatment.   The infant in this case is active and has normal respiratory and heart rates. The liver is enlarged, but this is a common finding in newborns with physiologic jaundice.

The absence of palpable spleen and normal feces are also consistent with physiologic jaundice.   Hemolytic disease of the newborn is a condition that occurs when the mother and fetus have incompatible blood types, leading to destruction of fetal red blood cells and an increase in bilirubin levels. This can cause severe jaundice and other complications. However, the infant in this case does not have any signs of hemolytic disease, such as pallor, tachycardia, or hepatosplenomegaly.  

Neonatal sepsis is a serious bacterial infection that can cause jaundice, but it usually presents with other symptoms such as fever, poor feeding, lethargy, and respiratory distress. Minkowski-Shauffard disease (also known as type 2 glycogen storage disease) is a rare metabolic disorder that can cause severe jaundice and liver disease in newborns, but it typically presents with other symptoms such as hypoglycemia and lactic acidosis.

Biliary atresia is a rare condition in which the bile ducts are blocked or absent, leading to jaundice and liver damage, but it usually presents with clay-colored stools and abnormal liver function tests.   In summary, the most probable diagnosis in this case is A. Physiologic jaundice, which is a common condition in newborns caused by increased bilirubin levels due to the breakdown of fetal red blood cells and delayed maturation of the liver’s ability to clear bilirubin.


166. A pediatrician talked to a mother of a 7 month old breast-fed boy and found out that the child was fed 7 times a day. How many times should the child of such age be fed?

A. 5 times

B. 3 times

C. 4 times

D. 6 times

E. 7 times


Answer:  5 times

 Explanation

As infants grow, their nutritional needs change, and the frequency of feedings may decrease. At 7 months of age, breast milk is still the primary source of nutrition, but infants may also be starting to eat solid foods. Breast-fed infants generally require fewer feedings per day than formula-fed infants, as breast milk is more easily digested.  

It is important for infants to receive frequent feedings to ensure they are getting enough nutrition and to promote healthy growth and development. However, feeding frequency can vary depending on the individual needs of the infant and the feeding habits of the mother.  

In general, breast-fed infants at 7 months of age should be fed 4 to 5 times per day, with each feeding lasting about 10 to 15 minutes per breast. If an infant is showing signs of hunger between feedings, additional feedings may be necessary. It is also important to monitor the infant’s growth, weight gain, and overall health to ensure they are getting enough nutrition.  

In summary, a breast-fed infant at 7 months of age typically requires 4 to 5 feedings per day, so the most appropriate answer is A. 5 times. However, feeding frequency can vary depending on the individual needs of the infant and the feeding habits of the mother.


167. In order to study impact of microclimate upon the human organism it is necessary to make systematic observation of air temperature over 3 days. Choose a device that will allow to make the most precise temperature records:

A. Thermograph

B. Alcohol thermometer

C. Mercury thermometer

D. August’s psychrometer

E. Assmann psychrometer


Answer: Thermograph

 Explanation

In order to make the most precise temperature records for the purpose of studying the impact of microclimate on the human organism, the most appropriate device would be A. Thermograph.   A thermograph is a type of thermometer that uses a pen and rotating drum to record temperature over time. It is a highly precise instrument that can make continuous temperature measurements with a high degree of accuracy.  

Alcohol and mercury thermometers are also precise instruments for measuring temperature, but they are not capable of making continuous measurements over time. They require manual readings to be taken at specific intervals, which may not capture the full range of temperature fluctuations.  

August’s and Assmann’s psychrometers are instruments used to measure relative humidity and dew point, not temperature. While these instruments can be useful for studying microclimate, they are not the most appropriate devices for measuring air temperature.  

In summary, for the purpose of studying the impact of microclimate on the human organism by making precise temperature records over time, the most appropriate device would be a thermograph, which is capable of making continuous temperature measurements with a high degree of accuracy.


168. A prematurely born girl is now 8 months old. She has dyspnea, tachycardia, hepatosplenomegaly, physical developmental lag, limb cyanosis. There is also parasternal cardiac hump, auscultation revealed systolodiastolic murmur in the II intercostal space on the left. AP is 90/0 mm Hg. What disease should be suspected?

A. Patent ductus arteriosus

B. Coarctation of aorta

C. Stenosis of aortal valve

D. Stenosis of pulmonary artery

E. Nonclosure of interventricular septum


Answer: Patent ductus arteriosus

 Explanation

PDA is a condition in which a blood vessel called the ductus arteriosus, which is present in fetuses and normally closes shortly after birth, remains open. This can cause increased blood flow to the lungs and lead to symptoms such as dyspnea, tachycardia, and cyanosis.

PDA can also cause hepatosplenomegaly and physical developmental lag.   The parasternal cardiac hump and systolodiastolic murmur in the II intercostal space on the left are consistent with PDA. The murmur is caused by the increased blood flow through the ductus arteriosus and can be heard throughout systole and diastole.

The limb cyanosis may be due to reduced blood flow to the extremities.   Coarctation of the aorta (B) is a narrowing of the aorta that can cause high blood pressure and reduced blood flow to the lower part of the body, but it would not cause the symptoms described in this case. Stenosis of the aortic valve (C) and pulmonary artery (D) are conditions that can cause murmurs, but they would not typically cause hepatosplenomegaly or limb cyanosis.

Nonclosure of the interventricular septum (E) is a type of congenital heart defect that can cause a murmur and other symptoms, but it would not typically cause hepatosplenomegaly or limb cyanosis.   In summary, the most likely diagnosis in this case is A. Patent ductus arteriosus, which is a condition in which a blood vessel called the ductus arteriosus remains open after birth, causing increased blood flow to the lungs and symptoms such as dyspnea, tachycardia, hepatosplenomegaly, and limb cyanosis. The parasternal cardiac hump and systolodiastolic murmur in the II intercostal space on the left are also consistent with PDA.


169. A 72 year old patient complains about pain and bleeding during defecation. Digital rectal investigation revealed a tumour of anal canal. After verification of the diagnosis the patient was diagnosed with squamous cell carcinoma. The secondary (metastatic) tumour will be most probably found in:

A. Lungs

B. Liver

C. Pelvic bones

D. Mediastinum

E. Brain


Answer:  Lungs

 Explanation

Squamous cell carcinoma is a type of cancer that can occur in many different parts of the body, including the anal canal. It is an aggressive cancer that can spread (metastasize) to other parts of the body.   The most common site of metastasis for anal canal squamous cell carcinoma is the lungs.

This is because the anal canal drains into the lymph nodes in the pelvic area, which then drain into the lymph nodes in the abdomen and chest. The cancer cells can travel through the lymphatic system to other parts of the body, including the lungs.   While squamous cell carcinoma can potentially metastasize to other organs such as the liver, pelvic bones, mediastinum, and brain, lung metastases are much more common.   In summary, in a patient with anal canal squamous cell carcinoma, the most likely site of metastasis is the lungs.

While metastasis to other organs is possible, lung metastases are much more common due to the drainage of the anal canal into the lymph nodes in the pelvic area, which then drain into the lymph nodes in the abdomen and chest.


170. A 52 year old patient complains about headache, weakness of his upper left extremity. Neurological symptoms become more intense during physical stress of the left extremity. Pulsation on the arteries of the left extremity is sharply dampened but it remains unchanged on the carotid arteries. What is the most probable diagnosis?

A. Occlusion of the left subclavicular artery, steal syndrome

B. Thoracal outlet syndrome

C. Raynaud’s syndrome

D. Takayasu’s disease

E. Occlusion of brachiocephalic trunk


Answer:  Occlusion of the left subclavicular artery, steal syndrome

 Explanation

Steal syndrome occurs when blood flow is diverted from one part of the body to another due to a blockage or narrowing of an artery. In this case, the blockage or narrowing is likely occurring in the left subclavian artery, which supplies blood to the left arm.

As a result, blood flow is diverted from the arm to the brain, causing neurological symptoms such as headache and weakness.   The pulsation on the arteries of the left extremity is sharply dampened, which is consistent with decreased blood flow to the arm.

However, the carotid arteries, which supply blood to the brain, remain unchanged, suggesting that blood flow to the brain is not affected.   Thoracic outlet syndrome (B) is a condition in which there is compression of the nerves or blood vessels in the thoracic outlet, which is the space between the collarbone and the first rib. This can cause symptoms such as weakness and numbness in the arm, but it would not cause a dampened pulse or affect blood flow to the brain.  

Raynaud’s syndrome (C) is a condition in which there is spasm of the blood vessels in the fingers and toes, causing them to turn white or blue and feel numb or cold. It would not cause symptoms such as headache or weakness in the arm.  

Takayasu’s disease (D) is a rare condition in which there is inflammation and narrowing of the large blood vessels in the body, including the subclavian artery. It can cause symptoms such as headache and arm weakness, but it would not cause steal syndrome or affect blood flow to the brain.  

Occlusion of the brachiocephalic trunk (E) would affect blood flow to both arms and the brain, not just the left arm, and it would not cause steal syndrome.   In summary, the most probable diagnosis in this case is A. Occlusion of the left subclavian artery, steal syndrome, which is a condition in which blood flow is diverted from the arm to the brain due to a blockage or narrowing of the subclavian artery.


171. A 25 year old woman applied to a maternity welfare clinic and complained about being unable to conceive within 3 years of regular sexual life. Examination revealed weight gain, male pattern of hair distribution on the pubis, excessive pilosis of thighs. Ovaries were dense and enlarged, basal temperature was monophase. What is the most probable diagnosis?

A. Sclerocystosis of ovaries

B. Tubo-ovaritis

C. Adrenogenital syndrome

D. Premenstrual syndrome

E. Gonadal dysgenesis


Answer: Sclerocystosis of ovaries

 Explanation

PCOS is a hormonal disorder that affects women of reproductive age. It is characterized by enlarged ovaries that contain multiple small cysts, as well as high levels of androgens (male hormones) and insulin resistance. The symptoms of PCOS can include weight gain, male pattern hair growth (hirsutism), and irregular periods.  

In this case, the patient is presenting with several symptoms that are consistent with PCOS, including weight gain, male pattern hair growth on the pubis, and excessive hair growth on the thighs. The dense and enlarged ovaries are also consistent with PCOS, as is the monophase basal temperature, which suggests a lack of ovulation.  
Tubo-ovaritis (B) is a condition in which there is inflammation of the fallopian tubes and ovaries, often due to a bacterial infection.

This would not typically cause the symptoms described in this case.   Adrenogenital syndrome (C) is a group of genetic disorders that affect the adrenal glands, leading to excessive production of androgens. This can cause symptoms such as hirsutism and irregular periods, but it would not typically cause enlarged ovaries or cysts.  

Premenstrual syndrome (D) is a condition that can cause a range of physical and emotional symptoms in the days leading up to menstruation, but it would not typically cause the symptoms described in this case.   Gonadal dysgenesis (E) is a condition in which the ovaries or testes do not develop properly, leading to infertility and other symptoms.

However, it would not typically cause the specific symptoms described in this case.   In summary, the most probable diagnosis in this case is A. Sclerocystic ovary syndrome (PCOS), which is a hormonal disorder characterized by enlarged ovaries containing multiple small cysts, as well as high levels of androgens and insulin resistance. The symptoms of PCOS can include weight gain, hirsutism, and irregular periods.


172. A woman consulted a therapeutist about fatigability, significant weight loss, weakness, loss of appetite. She has had amenorrhea for 8 months. A year ago she born a full-term child. Haemorrhage during labour made up 2 l. She got blood and blood substitute transfusions. What is the most probable diagnosis?

A. Sheehan’s syndrome

B. Stein-Leventhal syndrome

C. Shereshevsky-Turner’s syndrome

D. Homological blood syndrome

E. Vegetovascular dystonia


Answer:  Sheehan’s syndrome

 Explanation

Sheehan’s syndrome is a type of hypopituitarism that occurs as a result of severe postpartum hemorrhage. The loss of blood during childbirth can cause damage to the pituitary gland, which is located at the base of the brain and controls the production of several hormones.

In Sheehan’s syndrome, the pituitary gland is unable to produce enough hormones, leading to a range of symptoms such as fatigue, weight loss, weakness, and loss of appetite. Amenorrhea is also a common symptom.   In this case, the patient’s history of postpartum hemorrhage and blood transfusions is consistent with Sheehan’s syndrome. The fact that she has had amenorrhea for 8 months is also consistent with hormonal imbalances caused by pituitary dysfunction.  

Stein-Leventhal syndrome (B) is another name for polycystic ovary syndrome (PCOS), which is a hormonal disorder that affects women of reproductive age. While weight loss and loss of appetite can be symptoms of PCOS, amenorrhea is typically not present.  

Shereshevsky-Turner’s syndrome (C) is another name for Turner syndrome, which is a genetic disorder that affects females. It can cause a range of symptoms such as short stature, infertility, and heart defects, but it would not typically cause the specific symptoms described in this case.  

Homological blood syndrome (D) and vegetovascular dystonia (E) are not recognized medical conditions.   In summary, the most probable diagnosis in this case is A. Sheehan’s syndrome, which is a type of hypopituitarism that occurs as a result of severe postpartum hemorrhage. The symptoms include fatigue, weight loss, weakness, loss of appetite, and amenorrhea.


173. A 35 year old female patient suffering from cholelithiasis has broken her diet, and this caused an acute pain attack in the right subcostal are. The pain eased off on the third day, but the patient got progressing jaundice. What non-invasive diagnostic method should be applied?

A. Endoscopic retrograde cholangiopancreatography

B. Infusive cholecystocholangiography

C. Test for bilirubin

D. Duodenal probing

E. Survey radiography of abdominal organs


Answer:  Endoscopic retrograde cholangiopancreatography

 Explanation

Cholelithiasis (gallstones) can cause obstruction of the common bile duct, which can lead to jaundice and other complications. In this case, the patient’s acute pain attack and subsequent jaundice suggest that there may be an obstruction in the bile duct.   ERCP is a diagnostic and therapeutic procedure that is used to evaluate the bile ducts and pancreatic ducts.

It involves the use of an endoscope, which is a long, flexible tube with a camera and light at the end, to visualize the ducts and inject contrast dye to help identify any obstructions or abnormalities. ERCP is a non-invasive method that can be used to diagnose and treat certain conditions, such as cholelithiasis and bile duct obstruction.  

Infusive cholecystocholangiography (B) involves the injection of contrast dye into the gallbladder and bile ducts to visualize any obstructions or abnormalities. While it can be a useful diagnostic tool, it is more invasive than ERCP and may not be necessary in this case.   A test for bilirubin (C) can be used to measure the levels of bilirubin in the blood, which can help identify the cause of jaundice.

However, it is not a diagnostic tool to identify the underlying cause of the obstruction.   Duodenal probing (D) involves the insertion of a tube through the nose or mouth into the duodenum to obtain samples of bile for analysis. While it can be useful in certain cases, it is not as effective as ERCP in identifying the location and cause of an obstruction.  

Survey radiography of abdominal organs (E) can be used to visualize the abdominal organs and identify any abnormalities, but it may not be as useful in identifying the location and cause of an obstruction as ERCP.   In summary, in a patient with cholelithiasis and jaundice, the most appropriate non-invasive diagnostic method to identify any obstruction in the bile duct is A. Endoscopic retrograde cholangiopancreatography (ERCP).


174. A 68 year old patient complains about acute pain in his right foot, toe edema and darkening of skin of the IV toe. He has been suffering from diabetes mellitus for 15 years, doesn’t receive regular treatment. What complication of diabetes mellitus is it?

A. Gangrene of the IV toe on the right foot

B. Panaritium

C. Haematoma

D. Erysipelas

E. Fracture of the IV toe on the right foot


Answer: Gangrene of the IV toe on the right foot

 Explanation

Diabetes mellitus can lead to a range of complications, particularly if blood sugar levels are not well controlled over time. One of the most serious complications is peripheral vascular disease, which can lead to poor circulation and nerve damage in the extremities.

This can increase the risk of foot ulcers, infections, and ultimately gangrene, which is tissue death caused by a lack of blood flow.   In this case, the patient’s acute pain in the right foot, toe edema, and darkening of the skin of the IV toe are consistent with gangrene.

The fact that the patient has been suffering from diabetes mellitus for 15 years and is not receiving regular treatment increases the risk of complications.   Panaritium (B) is a bacterial infection of the tissue around a fingernail or toenail, but it would not typically cause the symptoms described in this case.   Haematoma (C) is a collection of blood outside of a blood vessel, often caused by an injury. It would not typically cause the symptoms described in this case.  

Erysipelas (D) is a skin infection caused by a type of bacteria, but it would not typically cause the symptoms described in this case.   Fracture of the IV toe (E) is a break in one of the bones in the toe, but it would not typically cause the symptoms described in this case.  

In summary, the most probable complication of diabetes mellitus in this case is A. Gangrene of the IV toe on the right foot, which is a serious condition caused by poor circulation and nerve damage in the extremities.

175. A plot of land with total area of 2,0 hectare was intended for building of a hospital. The maximal capacity of the hospital will be:

A. 100 beds

B. 200 beds

C. 400 beds

D. 800 beds

E. Over 1000 beds


Answer: 100 beds

 Explanation

The maximal capacity of the hospital that can be built on a 2.0-hectare plot of land cannot be accurately determined based solely on the size of the plot. The capacity of a hospital is dependent on a variety of factors, including the size and layout of the building, the availability of resources and staff, and the needs of the local population.  

However, as a rough estimate, a hospital with a capacity of 100 beds could be built on a 2.0-hectare plot of land. This would depend on a number of factors, such as the size and layout of the hospital building, the availability of infrastructure and resources, and the needs of the local population.   It’s important to note that the actual capacity of a hospital would need to be determined based on a detailed analysis of the specific location and needs of the community.


176. A 26 year old woman complains about edemata, swelling and painfulness of mammary glands, headache, tearfulness, irritability. These signs turn up 5 days before menstruation and disappear after its start. What clinical syndrome is it?

A. Premenstrual syndrome

B. Postcastration syndrome

C. Adrenogenital syndrome

D. Climacteric syndrome

E. Stein-Leventhal syndrome


Answer: Premenstrual syndrome

 Explanation

The maximal capacity of the hospital that can be built on a 2.0-hectare plot of land cannot be accurately determined based solely on the size of the plot. The capacity of a hospital is dependent on a variety of factors, including the size and layout of the building, the availability of resources and staff, and the needs of the local population.  

However, as a rough estimate, a hospital with a capacity of 100 beds could be built on a 2.0-hectare plot of land. This would depend on a number of factors, such as the size and layout of the hospital building, the availability of infrastructure and resources, and the needs of the local population.   It’s important to note that the actual capacity of a hospital would need to be determined based on a detailed analysis of the specific location and needs of the community.


177. A 27 year old patient suffers from haemophilia. He was admitted to the hospital with melena and skin pallor. Objectively: Ps- 110 bpm, AP- 100/60 mm Hg. In blood: Hb- 80 g/l, erythrocytes – 2, 8 · 1012/l. What medication should be administered in the first place?

A. Cryoprecipitate

B. Stored blood

C. Packed red blood cells

D. Dicinone

E. Epsilon-aminocapronic acid


Answer: Cryoprecipitate

 Explanation

Hemophilia is a genetic disorder that affects the blood’s ability to clot properly, which can lead to bleeding episodes. In this case, the patient’s symptoms of melena and skin pallor suggest that he is experiencing significant bleeding.  

Cryoprecipitate is a blood product that contains concentrated clotting factors, including factor VIII and von Willebrand factor. It is used to treat bleeding episodes in patients with hemophilia. Cryoprecipitate is typically administered intravenously and can help to stop bleeding by replacing the missing clotting factors.  

Stored blood (B) is not the most appropriate medication for this patient because it does not contain the specific clotting factors that are missing in hemophilia.   Packed red blood cells (C) are also not the most appropriate medication for this patient because they do not contain the clotting factors needed to stop bleeding in hemophilia.  

Dicinone (D) is a medication that is used to promote blood clotting by increasing the production of clotting factors. However, it would not be the most appropriate medication to administer in the first place to a patient with hemophilia who is experiencing active bleeding.  

Epsilon-aminocapronic acid (E) is another medication that can be used to treat bleeding in patients with hemophilia. It works by preventing the breakdown of blood clots. However, cryoprecipitate would be the most appropriate medication to administer in the first place to this patient with hemophilia and signs of bleeding.  

In summary, the most appropriate medication to administer in the first place to a patient with hemophilia and signs of bleeding (such as melena and skin pallor) is A. Cryoprecipitate, which contains concentrated clotting factors that can help to stop bleeding in hemophilia.


178. A 38 year old patient complains about inertness, subfebrile temperature, enlargement of lymph nodes, nasal haemorrhages, bone pain. Objectively: the patient’s skin and mucous membranes are pale, palpation revealed enlarged painless lymph nodes; sternalgia; liver was enlarged by 2 cm, spleen – by 5 cm, painless. In blood: erythrocytes – 2, 7 · 1012/l, Hb- 84 g/l, leukocytes – 58 · 109/l, eosinophils – 1%, stab neutrophils – 2%, segmented neutrophils – 12%, lymphocytes – 83%, lymphoblasts – 2%, smudge cells; ESR57 mm/h. What is the most probable diagnosis?

A. Chronic lymphatic leukemia

B. Chronic myeloleukemia

C. Acute lymphatic leukemia

D. Acute myeloleukemia

E. Lymphogranulomatosis


Answer:  Chronic lymphatic leukemia

 Explanation

Chronic lymphocytic leukemia (CLL) is a type of cancer that affects the white blood cells, specifically the lymphocytes. It typically presents with symptoms such as fatigue, enlarged lymph nodes, and organ enlargement.

The laboratory findings in this case, including anemia (low hemoglobin), low red blood cell count, and high white blood cell count with an increased percentage of lymphocytes and smudge cells, are also consistent with CLL.   Chronic myeloid leukemia (B) is a type of cancer that affects the white blood cells, specifically the myeloid cells. It typically presents with an enlarged spleen, fatigue, and weight loss. However, the laboratory findings in this case are not consistent with CML.  

Acute lymphocytic leukemia (C) is a type of cancer that affects the white blood cells, specifically the lymphocytes. It typically presents with symptoms such as fatigue, bone pain, and fever. However, the high percentage of lymphocytes and the chronicity of the symptoms are not consistent with ALL.  

Acute myeloid leukemia (D) is a type of cancer that affects the white blood cells, specifically the myeloid cells. It typically presents with symptoms such as fatigue, fever, and weight loss. However, the laboratory findings in this case are not consistent with AML.  

Lymphogranulomatosis (E), also known as Hodgkin’s lymphoma, is a type of cancer that affects the lymphatic system. It typically presents with symptoms such as enlarged lymph nodes, fatigue, and weight loss. However, the laboratory findings in this case are not consistent with Hodgkin’s lymphoma.   In summary, the most probable diagnosis for this patient’s symptoms and laboratory findings is A. Chronic lymphatic leukemia.


179. A 36 year old female patient complains about general weakness, edemata of her face and hands, rapid fatigability during walking, difficult diglutition, cardiac irregularities. These symptoms turned up 11 days after holiday at the seaside. Objectively: face erythema, edema of shin muscles. Heart sounds are muffled, AP is 100/70 mm Hg. In blood: ASAT activity is 0,95 millimole/h·l, ALAT – 1,3 micromole/h·l, aldolase – 9,2 IU/l, creatine phosphokinase – 2,5 micromole Р/g·l. What method of study would be the most specific?

A. Muscle biopsy

B. ECG C. Echocardiogram

D. Electromyography

E. Determination of cortisol concentration in blood and urine


Answer:  Muscle biopsy

 Explanation

The patient’s symptoms of weakness, edema, and cardiac irregularities suggest a possible diagnosis of myopathy, which is a disorder that affects the muscles. The laboratory findings of elevated creatine phosphokinase and aldolase levels also support this diagnosis.  

Muscle biopsy is the most specific method of study for diagnosing myopathy. It involves taking a small sample of muscle tissue for examination under a microscope. This can help to identify any abnormalities in the muscle tissue, such as inflammation, necrosis, or fiber type changes, that could be causing the patient’s symptoms.  

ECG (B) and echocardiogram (C) are useful diagnostic tools for assessing heart function, but they may not be specific enough for diagnosing a myopathy.   Electromyography (D) is a diagnostic test that measures the electrical activity of muscles and nerves. While it can provide useful information about the function of the patient’s muscles, it may not be specific enough for diagnosing a myopathy.  

Determination of cortisol concentration in blood and urine (E) is a test that can be used to diagnose disorders of the adrenal gland, such as Cushing’s syndrome. While it can provide useful information about the patient’s hormonal balance, it is not specific for diagnosing a myopathy.  

In summary, the most specific method of study for diagnosing a possible myopathy in this patient is A. Muscle biopsy, which can help to identify any abnormalities in the muscle tissue that could be causing the patient’s symptoms.


180. A 33 year old patient has acute blood loss (erythrocytes – 2, 2·1012/l, Hb- 55 g/l), blood group is A(II)Rh+. Accidentally the patient got transfusion of donor packed red blood cells of AB(IV )Rh+ group. An hour later the patient became anxious, got abdominal and lumbar pain. Ps- 134 bpm, AP- 100/65 mm Hg, body temperature – 38, 6oC. After catheterization of urinary bladder 12 ml/h of dark-brown urine were obtained. What complication is it?

A. Acute renal insufficiency

B. Cardial shock

C. Allergic reaction to the donor red blood cells

D. Citrate intoxication

E. Toxic infectious shock


Answer: Acute renal insufficiency

 Explanation

The patient received a transfusion of packed red blood cells of a different blood group than their own. This can lead to a transfusion reaction, which is an adverse reaction to a blood transfusion. The symptoms of anxiety, abdominal and lumbar pain, increased heart rate, low blood pressure, and fever suggest a transfusion reaction.  

The dark-brown urine and low urine output (12 ml/h) suggest that the patient is experiencing acute renal insufficiency, which is a serious complication of a transfusion reaction. Acute renal insufficiency can occur as a result of the release of hemoglobin and other breakdown products from the transfused red blood cells, which can cause damage to the kidneys.  

Cardial shock (B) is a type of shock that occurs when the heart is unable to pump enough blood to meet the body’s needs. This is not the most likely complication in this case, as the patient’s symptoms are more consistent with a transfusion reaction and acute renal insufficiency.   Allergic reaction to the donor red blood cells (C) is another possible complication of a transfusion reaction. However, the dark-brown urine and low urine output suggest that the patient is experiencing acute renal insufficiency.  

Citrate intoxication (D) is a possible complication of a blood transfusion, as citrate is used as an anticoagulant to prevent the blood from clotting during storage.

However, citrate intoxication typically causes symptoms such as hypocalcemia and metabolic alkalosis, which are not present in this case.   Toxic infectious shock (E) is a type of shock that occurs as a result of a severe infection.

This is not the most likely complication in this case, as the patient’s symptoms are more consistent with a transfusion reaction and acute renal insufficiency.   In summary, the most likely complication in this patient who received a transfusion of packed red blood cells of a different blood group is A. Acute renal insufficiency, which can occur as a result of a transfusion reaction and can lead to damage to the kidneys.


181. A parturient woman is 27 year old, it was her second labour, delivery was at term, normal course. On the 3rd day of postpartum period body temperature is 36, 8oC, Ps – 72/min, AP – 120/80 mm Hg. Mammary glands are moderately swollen, nipples are clean. Abdomen is soft and painless. Fundus of uterus is 3 fingers below the umbilicus. Lochia are bloody, moderate. What is the most probable diagnosis?

A. Physiological course of postpartum period

B. Subinvolution of uterus

C. Postpartum metroendometritis

D. Remnants of placental tissue after labour

E. Lactostasis


Answer: Physiological course of postpartum period

 Explanation

The patient is a 27-year-old woman who has just given birth to her second child. Her delivery was at term and had a normal course. On the third day of the postpartum period, her body temperature is slightly elevated at 36.8°C, but her pulse rate and blood pressure are within normal limits. Her mammary glands are moderately swollen, and her abdomen is soft and painless. The fundus of her uterus is 3 fingers below the umbilicus, which is also within the normal range for this stage of the postpartum period. Her lochia are described as moderate.  

All of the clinical findings described in this case are consistent with the physiological course of the postpartum period. In the immediate postpartum period, the uterus undergoes involution, which is the process of returning to its pre-pregnancy size and shape. The uterus normally contracts and reduces in size, and the lochia gradually change from a red, bloody discharge to a yellowish-white discharge as healing progresses. The mammary glands also undergo changes as lactation begins.  

Subinvolution of the uterus (B), or failure of the uterus to return to its pre-pregnancy size, can occur in some cases and can lead to persistent vaginal bleeding and other complications. However, the clinical findings in this case are not consistent with subinvolution of the uterus.   Postpartum metroendometritis (C) is an infection of the uterus that can occur after delivery. However, the absence of abdominal pain, tenderness, and foul-smelling discharge makes this diagnosis less likely.  

Remnants of placental tissue after labor (D) can lead to postpartum bleeding and infection. However, the clinical findings in this case are not consistent with retained placental tissue.   Lactostasis (E) is a condition in which milk accumulates in the breast and can lead to engorgement and discomfort.

However, the clinical findings in this case are consistent with normal mammary gland changes during the postpartum period.   In summary, the most probable diagnosis for this patient’s clinical findings on the third day of the postpartum period is A. Physiological course of postpartum period.


182. A patient suffering from gastroesophageal reflux has taken from time to time a certain drug that “reduces acidity”for 5 years. This drug was recommended by a pharmaceutist. The following side effects are observed: osteoporosis, muscle weakness, indisposition. What drug has such following effects?

A. Aluminium-bearing antacid

B. Inhibitor of proton pump

C. 2-blocker

D. Metoclopramide

E. Gastrozepin


Answer:  Aluminium-bearing antacid

 Explanation

Aluminium-containing antacids, such as aluminum hydroxide, are commonly used to treat gastroesophageal reflux disease (GERD) by reducing the acidity in the stomach. However, long-term use of these antacids can lead to several side effects, including osteoporosis, muscle weakness, and malaise.  

Osteoporosis is a condition characterized by weakened bones, which can increase the risk of fractures. The use of aluminum-containing antacids has been associated with an increased risk of osteoporosis due to the fact that aluminum can inhibit the absorption of calcium and other minerals in the body.  

Muscle weakness and malaise are also common side effects of aluminum-containing antacids. This is thought to be due to the fact that aluminum can accumulate in the body, leading to toxic effects on the muscles and nerves.  

Inhibitors of proton pump (B), such as omeprazole and esomeprazole, are another class of drugs commonly used to treat GERD. While they may have side effects, such as headache and diarrhea, they are not typically associated with the side effects described in this case.  

2-blockers (C), such as ranitidine and cimetidine, are another class of drugs used to treat GERD. While they may have side effects, such as dizziness and headache, they are not typically associated with the side effects described in this case.   Metoclopramide (D) is a medication used to treat nausea and vomiting. While it may have side effects, such as drowsiness and restlessness, it is not typically used to treat GERD and is unlikely to be responsible for the side effects described in this case.  

Gastrozepin (E) is not a commonly used medication and is not typically used to treat GERD. Therefore, it is unlikely to be responsible for the side effects described in this case.   In summary, the drug most likely responsible for the observed side effects of osteoporosis, muscle weakness, and malaise in a patient with gastroesophageal reflux disease is A. Aluminum-bearing antacid.


183. A 34 year old patient complains of profuse sweating at night, skin itching, weight loss (9 kg within the last 3 months). Examination revealed malnutrition, skin pallor. Palpation of neck and inguinal areas revealed dense elastic lymph nodes for about 1 cm in diameter, nonmobile, non-adhering to skin. What is the most probable diagnosis?

A. Lymphogranulomatosis

B. Chronic lymphadenitis

C. Lymphosarcoma

D. Burkitt’s lymphoma

E. Cancer metastases


Answer:  Lymphogranulomatosis

 Explanation

The patient’s symptoms of profuse sweating at night, skin itching, and weight loss, along with the presence of dense, non-mobile lymph nodes, suggest a diagnosis of lymphoma. Hodgkin’s lymphoma is a type of lymphoma that often presents with these symptoms and can cause the enlargement of lymph nodes in the neck, armpit, and groin areas.   The skin pallor and malnutrition may also be indicative of an underlying malignancy such as lymphoma.

The lymph nodes in lymphogranulomatosis are typically firm, non-mobile, and non-adherent to skin or underlying tissues.   Chronic lymphadenitis (B) is a condition characterized by the long-term inflammation of lymph nodes. While it can cause lymph node enlargement, it is typically not associated with the other symptoms described in this case.  

Lymphosarcoma (C) is a term that was previously used to describe non-Hodgkin’s lymphoma, which is a type of lymphoma that is not characterized by the presence of Reed-Sternberg cells. However, the patient’s symptoms and lymph node characteristics are more consistent with Hodgkin’s lymphoma.  

Burkitt’s lymphoma (D) is a type of non-Hodgkin’s lymphoma that is characterized by the presence of rapidly growing tumors. While it can cause lymph node enlargement and other symptoms, it is typically not associated with skin itching and night sweats.  

Cancer metastases (E) occur when cancer cells from one part of the body spread to other parts of the body. While this is a possibility, the presence of dense, non-mobile lymph nodes is more suggestive of a primary lymphoid malignancy, such as lymphogranulomatosis.   In summary, the most probable diagnosis for this patient’s symptoms of profuse sweating at night, skin itching, weight loss, and the presence of dense, non-mobile lymph nodes is A. Lymphogranulomatosis, also known as Hodgkin’s lymphoma.


184. A department chief of an in-patient hospital is going to examine resident doctors as to observation of medicaltechnological standards of patient service. What documentation should be checked for this purpose?

A. Health cards of in-patients

B. Statistic cards of discharged patients

C. Treatment sheets

D. Registry of operative interventions

E. Annual report of a patient care institution


Answer:  Health cards of in-patients

 Explanation

Health cards of in-patients contain detailed information about the patient’s medical history, physical examination, diagnostic tests, treatment plan, and progress notes. By reviewing health cards, the department chief can evaluate whether the resident doctors are following established medical technological standards of patient service.  

The health card includes information about the patient’s current condition, including vital signs, medications, and any procedures or interventions performed. It also includes information about the patient’s medical history, such as past illnesses, surgeries, and allergies. The health card is updated regularly and provides a comprehensive record of the patient’s care during their hospitalization.   Statistic cards of discharged patients (B) provide information about the number of patients discharged from the hospital and their diagnoses.

While this information may be useful, it does not provide detailed information about individual patient care or adherence to medical technological standards.   Treatment sheets (C) provide a detailed record of the patient’s treatment plan and progress notes during their hospitalization. However, they may not provide a comprehensive picture of the patient’s care, as they may not include information about diagnostic tests or consultations with specialists.  

The registry of operative interventions (D) provides information about surgical procedures performed on patients. While this information may be useful for evaluating the quality of surgical care, it does not provide information about non-surgical aspects of patient care.  

The annual report of a patient care institution (E) provides a summary of the institution’s activities, including the number of patients treated, types of services provided, and financial information. While this information may be useful for evaluating the overall performance of the institution, it does not provide detailed information about individual patient care or adherence to medical technological standards.   In summary, the documentation that should be checked to observe medical technological standards of patient service is A. Health cards of in-patients.


185. A woman is 34 years old, it is her tenth labor at full term. It is known from the anamnesis that the labor started 11 hours ago, labor was active, painful contractions started after discharge of waters and became continuous. Suddenly the parturient got knife-like pain in the lower abdomen and labor activity stopped. Examination revealed positive symptoms of peritoneum irritation, illdefined uterus outlines. Fetus was easily palpable, movable. Fetal heartbeats wasn’t auscultable. What is the most probable diagnosis?

A. Rupture of uterus

B. Uterine inertia

C. Discoordinated labor activity

D. Risk of uterus rupture

E. II labor period


Answer:  Rupture of uterus

 Explanation

The patient is a 34-year-old woman in her tenth labor at full term. The labor had been active for 11 hours, but suddenly stopped after the patient experienced knife-like pain in the lower abdomen. Examination revealed positive symptoms of peritoneum irritation, ill-defined uterus outlines, and the absence of fetal heartbeats.

These findings suggest a diagnosis of uterine rupture.   Uterine rupture is a rare but serious complication of labor that can occur when the uterus tears or separates along the scar of a previous cesarean section or other surgical procedures, or due to trauma, such as excessive force during contractions or the use of instruments during delivery.

It can be life-threatening for both the mother and the fetus.   The positive symptoms of peritoneum irritation, such as abdominal tenderness and guarding, are indicative of a rupture of the uterus. The ill-defined uterus outlines suggest that the uterus has lost its normal shape and may not be contracting effectively.

The absence of fetal heartbeats indicates that the fetus may have died due to the rupture.   Uterine inertia (B) is a condition in which the uterus fails to contract effectively during labor. This can lead to prolonged labor and other complications, but it is not typically associated with sudden abdominal pain or peritoneal irritation.  

Discoordinated labor activity (C) is a condition in which the contractions of the uterus are not coordinated and effective in moving the fetus down the birth canal. This can lead to a prolonged labor, but it is not typically associated with sudden abdominal pain or peritoneal irritation.  

Risk of uterine rupture (D) is a concern in patients who have had previous cesarean sections or other surgical procedures on the uterus. While this patient has had multiple previous pregnancies, the sudden onset of abdominal pain and peritoneal irritation suggest that a rupture may have already occurred.  

The second stage of labor (E), or the pushing stage, typically occurs after the cervix is fully dilated and the baby is ready to be delivered. It is not typically associated with sudden abdominal pain or peritoneal irritation.   In summary, the most probable diagnosis for this patient’s sudden onset of abdominal pain, peritoneal irritation, and absence of fetal heartbeats during labor is A. Rupture of uterus.


186. A 22 year old female patient complains about frequent and painful urination, urge to urinate at night, enuresis, pain in the suprapubic and lumbar area. Her urine often has beer colouring. She got married a month ago. Objectively: general state is satisfactory. Lung examination revealed vesicular respiration. Heart sounds are rhythmic, heart rate is 78/min, AP- 128/68 mm Hg. Abdomen is soft, painful in the suprapubic area. Urine contains 12-18 erythrocytes and 12-15 bacteria within eyeshot. What is the most probable diagnosis?

A. Infection of inferior urinary tracts – cystitis

B. Urolithiasis

C. Infection of superior urinary tracts – pyelonephritis

D. Gonorrhoea E. Primary syphilis

Answer:  Infection of inferior urinary tracts – cystitis

 Explanation

The patient’s symptoms of frequent and painful urination, urge to urinate at night, enuresis, and pain in the suprapubic and lumbar area are all indicative of a urinary tract infection (UTI), specifically cystitis, which is an infection of the bladder. The presence of erythrocytes and bacteria in the urine also supports this diagnosis.  

Cystitis is a common UTI that occurs when bacteria, usually Escherichia coli, enter the bladder and cause inflammation. It can cause pain and discomfort during urination, as well as frequent and urgent urination. The presence of blood in the urine, indicated by the beer color, is also a common symptom of cystitis.  

The patient’s general state is satisfactory, and lung and heart examinations reveal normal findings, indicating that the infection has not spread to other parts of the body.   Urolithiasis (B) is a condition in which stones or crystals form in the urinary tract.

While it can cause similar symptoms to cystitis, such as pain during urination and in the suprapubic and lumbar area, it is typically associated with more severe pain and may not cause frequent urination or enuresis.  

Infection of superior urinary tracts – pyelonephritis (C) is a more serious UTI that occurs when bacteria travel from the bladder to the kidneys. It can cause fever, chills, and flank pain in addition to urinary symptoms. However, the absence of these symptoms and normal examination findings make this diagnosis less likely.  

Gonorrhea (D) and primary syphilis (E) are sexually transmitted infections that can cause similar symptoms to cystitis, such as painful urination and discharge. However, the absence of discharge and the presence of erythrocytes and bacteria in the urine make these diagnoses less likely.   In summary, the most probable diagnosis for this patient’s symptoms of painful and frequent urination, enuresis, suprapubic and lumbar pain, and beer-colored urine is A. Infection of inferior urinary tracts – cystitis.


187. Examination of placenta revealed a defect. An obstetrician performed manual investigation of uterine cavity, uterine massage. Prophylaxis of endometritis in the postpartum period should involve following actions:

A. Antibacterial therapy

B. Instrumental revision of uterine cavity

C. Haemostatic therapy

D. Contracting agents

E. Intrauterine instillation of dioxine


Answer: Antibacterial therapy

 Explanation

Endometritis is an infection of the lining of the uterus that can occur after delivery. It is more common in patients who have had a placental defect or manual investigation of the uterine cavity, as these procedures can introduce bacteria into the uterus.  

Antibacterial therapy is the most important prophylactic measure for endometritis in the postpartum period. It is typically given prophylactically to all patients who have had a cesarean section or other invasive procedures during delivery, as well as those with risk factors for infection. Antibiotics can help to prevent bacterial growth and reduce the risk of endometritis.   Instrumental revision of the uterine cavity (B) may be necessary if there is retained placenta or other tissue in the uterus.

However, it is not typically used as a prophylactic measure for endometritis.   Haemostatic therapy (C) and contracting agents (D) may be used to control bleeding after delivery, but they are not typically used as prophylactic measures for endometritis.  


Intrauterine instillation of dioxin (E) is not a recommended treatment or prophylactic measure for endometritis. It may have serious side effects and is not commonly used in obstetrics.   In summary, the prophylaxis of endometritis in the postpartum period following a placental defect and manual investigation of the uterine cavity should involve A. Antibacterial therapy. Other measures may be necessary depending on the specific circumstances, but antibiotics are the most important prophylactic measure for preventing postpartum endometritis.


188. A 9 year old boy had acute respiratory viral infection. After it there appeared polydipsia, polyuria, weakness, nausea. Examination revealed the following symptoms: mental confusion, dry skin, soft eyeballs, Kussmaul’s respiration, acetone smell from the mouth, muffled heart sounds, soft and painless abdomen. Blood sugar was 19 millimole/l. What acute condition is it?

A. Ketoacidotic coma

B. Hyperosmolar coma

C. Cerebral coma

D. Hepatic coma

E. Acute renal insufficiency


Answer: Ketoacidotic coma

 Explanation

The patient is a 9-year-old boy who had an acute respiratory viral infection. He developed symptoms of polydipsia, polyuria, weakness, and nausea, which are suggestive of hyperglycemia. Examination revealed mental confusion, dry skin, soft eyeballs, Kussmaul’s respiration, acetone smell from the mouth, muffled heart sounds, and a soft and painless abdomen. Blood sugar was elevated at 19 millimoles/liter.  

These findings are consistent with diabetic ketoacidosis (DKA), which is a serious complication of diabetes mellitus that can occur when the body produces high levels of blood acids called ketones. It is commonly seen in patients with type 1 diabetes, but can also occur in patients with type 2 diabetes.  

The symptoms of DKA include hyperglycemia, dehydration, electrolyte imbalances, and acidosis. The acetone smell from the mouth is due to the presence of ketones, which are byproducts of fat metabolism that the body produces when it cannot use glucose for energy. Kussmaul’s respiration is a deep and rapid breathing pattern that the body uses to try to eliminate excess carbon dioxide and decrease the acidity of the blood.  

Mental confusion and muffled heart sounds can occur due to electrolyte imbalances and acidosis. Soft and painless abdomen is a sign of decreased bowel sounds, which can occur with electrolyte imbalances and dehydration.   Hyperosmolar coma (B) is another complication of diabetes mellitus that can occur when blood sugar levels are very high, but it is typically seen in patients with type 2 diabetes and is characterized by severe dehydration and hyperosmolarity, rather than acidosis.  

Cerebral coma (C) is a general term used to describe a state of unconsciousness due to brain dysfunction. It is not specific to any particular condition.   Hepatic coma (D) is a complication of liver disease in which the liver is unable to remove toxins from the blood, leading to a buildup of ammonia and other substances in the blood. This is not likely in this case, as the patient’s liver function has not been reported to be impaired.  

Acute renal insufficiency (E) is a condition in which the kidneys suddenly stop working properly. While it can cause electrolyte imbalances and acidosis, it is not typically associated with hyperglycemia.   In summary, the most probable acute condition in this case is A. Ketoacidotic coma, which is a serious complication of diabetes mellitus that can occur when the body produces high levels of blood acids called ketones.


189. A patient consulted a venereologist about painful urination, reddening of the external opening of urethra, profuse purulent discharges from the urethra. He considers himself to be ill for 3 days. He also associates the disease with a casual sexual contact that took place for about a week ago. If provisional diagnosis “acute gonorrheal urethritis”will be confirmed, then bacteriological study of urethral discharges will reveal:

A. Gram-negative diplococci

B. Gram-positive diplococci

C. Spirochaete

D. Proteus vulgaris

E. Mycoplasma

Answer: Gram-negative diplococci


 Explanation

Gram-negative diplococci. Acute gonorrheal urethritis is a sexually transmitted infection caused by the bacterium Neisseria gonorrhoeae, which is a gram-negative diplococcus. The symptoms described by the patient are consistent with gonorrheal urethritis, including painful urination, reddening of the external opening of the urethra, and purulent discharge from the urethra.

A definitive diagnosis of gonorrhea can be made by identifying N. gonorrhoeae in the urethral discharge using bacteriological testing, such as a Gram stain and culture


190. A 30 year old patient complains about inability to become pregnant over 3 years of married life. The patient is of supernutrition type, she has hair along the median abdominal line, on the internal thigh surface and in the peripapillary area. Menses started at the age of 16, they are infrequent and non-profuse. US revealed that the uterus was of normal size, ovaries were 4х5х5 cm large and had a lot of cystic inclusions. What is the most probable diagnosis?

A. Polycystic ovaries

B. Ovarian cystoma

C. Chronic oophoritis

D. Menstrual irregularity

E. Bilateral ovarian tumours

Answer: Polycystic ovaries

 Explanation

The patient’s symptoms and ultrasound findings suggest polycystic ovary syndrome (PCOS). PCOS is a common hormonal disorder in women of reproductive age, characterized by high levels of male hormones (androgens), infrequent or absent ovulation, and multiple small cysts on the ovaries.  

The patient’s history of menstrual irregularity, including infrequent and non-profuse periods, is consistent with PCOS. Additionally, the presence of excess hair growth (known as hirsutism) on the abdomen, inner thighs, and around the nipples (peripapillary area) is a common symptom of PCOS due to the increased androgen levels.  

The ultrasound findings of enlarged ovaries with multiple cystic inclusions further support the diagnosis of polycystic ovaries. However, it is important to note that not all women with PCOS have cysts on their ovaries, and not all women with ovarian cysts have PCOS.  

Treatment for PCOS often involves lifestyle modifications, such as weight loss and exercise, as well as medications to regulate menstrual cycles, reduce androgen levels, and induce ovulation if pregnancy is desired.


191. On the second day after preventive vaccination a 2 year old boy got abdominal pain without clear localization, body temperature rose up to 38oC. On the third day the child got red papular haemorrhagic eruption on the extensor surfaces of limbs and around the joints. Knee joints were edematic and slightly painful. Examination of other organs and systems revealed no pathological changes. What is the most probable diagnosis?

A. Haemorrhagic vesiculitis

B. Thrombocytopenic purpura

C. Meningococcemia

D. Urticaria

E. DIC syndrome


Answer: Haemorrhagic vesiculitis

 Explanation

The child’s symptoms of abdominal pain, fever, and a rash suggest a diagnosis of haemorrhagic vesiculitis. This condition is a rare adverse reaction to certain vaccines, including the smallpox vaccine, and is characterized by fever, abdominal pain, and a rash that typically starts on the extremities and spreads to the trunk. The rash consists of red, papular, and hemorrhagic lesions that may evolve into vesicles or pustules.  

Thrombocytopenic purpura (B) is another condition that can cause bleeding and a rash, but it is not typically associated with abdominal pain and fever. Meningococcemia (C) is a bacterial infection that can cause fever, a rash, and joint pain, but it is less likely in the absence of other signs of infection.

Urticaria (D) is a skin reaction characterized by raised, itchy wheals, but it does not typically cause abdominal pain or fever. Disseminated intravascular coagulation (E) is a serious condition that can be caused by various factors, including infection, but it is less likely in the absence of other signs of bleeding or thrombosis.  

It is important to note that adverse reactions to vaccines are rare, and the benefits of vaccination usually outweigh the risks. If a child experiences symptoms after vaccination, it is important to seek medical attention promptly to determine the cause and appropriate treatment.


192. On the 6th day of life a child got multiple vesicles filled with seropurulent fluid in the region of occiput, neck and buttocks. General condition of the child is normal. What disease should be suspected?

A. Vesiculopustulosis

B. Impetigo neonatorum

C. Miliaria

D. Impetigo

E. Epidermolysis bullosa


Answer:  Vesiculopustulosis

 Explanation

Vesiculopustulosis is a benign, self-limited condition that affects newborns within the first few weeks of life. It is characterized by the development of multiple, small vesicles or pustules filled with serous or seropurulent fluid, typically located on the scalp, neck, and buttocks.

The lesions are usually asymptomatic and resolve spontaneously within a few days to weeks.   Impetigo neonatorum (B) is a bacterial skin infection that can occur in newborns and is characterized by the formation of honey-colored crusts on the skin.

Miliaria (C) is a heat rash that can occur in newborns and is characterized by small, red, raised bumps on the skin. Impetigo (D) is a bacterial skin infection that can affect individuals of any age and is characterized by the formation of honey-colored crusts on the skin.

Epidermolysis bullosa (E) is a genetic disorder that affects the skin and mucous membranes, causing blistering and skin fragility.   In this case, the symptoms and location of the lesions are consistent with vesiculopustulosis, a benign condition that does not require specific treatment. However, if the lesions become infected or do not resolve within a few weeks, further evaluation and treatment may be necessary.


193. A 4 month old child was admitted to a surgical department 8 hours after the first attack of anxiety. The attacks happen every 10 minutes and last for 2-3 minutes, there was also one-time vomiting. Objectively: the child’s condition is grave. Abdomen is soft, palpation reveals a tumour-like formation in the right iliac area. After rectal examination the doctor’s finger was stained with blood. What is the most probable diagnosis?

A. Ileocecal invagination

B. Gastrointestinal haemorrhage

C. Wilm’s tumour

D. Helminthic invasion

E. Pylorostenosis


Answer: Ileocecal invagination

 Explanation

Ileocecal invagination is a serious condition that occurs when a portion of the intestine folds into itself, causing a blockage of the intestinal lumen. The symptoms typically include colicky abdominal pain, vomiting, and bloody stools.

The pain may be intermittent and occur in waves, as described in this case, and the child’s condition may progressively worsen over time.   The presence of a tumour-like formation in the right iliac area and rectal bleeding are both consistent with ileocecal invagination.

The tumour-like formation may be the invaginated portion of the intestine, which can be palpable on physical examination. Rectal bleeding may occur due to the pressure exerted on the mucosa of the intestine during invagination.   Gastrointestinal hemorrhage (B) can also cause bloody stools and vomiting, but it is less likely to cause a tumour-like formation and colicky abdominal pain.

Wilm’s tumour (C) is a type of childhood kidney cancer and is not associated with the symptoms described in this case. Helminthic invasion (D) can cause abdominal pain and vomiting, but it is less likely to cause rectal bleeding and a tumour-like formation.

Pyloric stenosis (E) is a condition that affects infants and is characterized by projectile vomiting, but it does not typically cause colicky abdominal pain or rectal bleeding.   Immediate medical attention is necessary in cases of ileocecal invagination to prevent bowel necrosis and other complications. Treatment may involve air or barium enema, surgical reduction, or surgery to remove the affected portion of the intestine.


194. Estimation of physical development of a child involved dynamometry and estimation of body weight and length, annual gain in body length, chest circumference, number of permanent teeth, secondary sexual characters, lung vital capacity. Which of the mentioned indices relates to the physiometric ones?

A. Lung vital capacity, dynamometry

B. Body length and weight, chest circumference

C. Secondary sexual characters

D. Number of permanent teeth

E. Annual gain in body length


Answer: Lung vital capacity, dynamometry

 Explanation

Dynamometry is a method of measuring muscle strength and is used to assess physical development in children. Lung vital capacity is a measure of lung function and can be used to evaluate respiratory development. Both of these indices are considered physiometric, as they involve the measurement of physiological parameters.  

Body length and weight, chest circumference, and annual gain in body length (B and E) are anthropometric measures, which involve the measurement of physical dimensions and growth rates. These measures are useful for assessing overall growth and development in children and can be used to monitor progress over time.  

Secondary sexual characters (C) are developmental changes that occur during puberty, including the growth of pubic hair, breast development, and testicular enlargement. These changes are not useful for assessing physical development in younger children.  

The number of permanent teeth (D) is a measure of dental development and is not typically used to assess overall physical development. However, dental development can provide important information about a child’s age and growth trajectory.


195. During preventive examination a 16 year old patient presented no problems. Objectively: the patient has signs of malnutrition, he is asthenic, AP is 110/70 mm Hg, Ps is 80 bpm, cardiac border is normal, auscultation above the cardiac apex reveals three sounds, cardiac murmur is absent. ECG shows no pathological changes, phonocardiogram shows that the third sound comes 0,15 s after the second one above the apex. How are these changes called?

A. III physiological sound

B. Fout-ta-ta-rou (reduplication of the 2nd sound)

C. Protodiastolic gallop rhythm

D. Atrial gallop rhythm

E. IV physiological sound


Answer: III physiological sound

 Explanation

The third heart sound (S3) is a low-frequency sound that occurs in early diastole and is associated with rapid filling of the ventricles. It is a normal physiological sound in children and young adults, and can be heard in some healthy individuals.

The S3 sound is sometimes referred to as a “ventricular gallop” or “protodiastolic gallop” due to its timing in the cardiac cycle.   In this case, the presence of a third heart sound on auscultation and its timing on the phonocardiogram (0.15 seconds after the second sound) are consistent with a normal physiological S3 sound.

The absence of a cardiac murmur, normal ECG, and normal blood pressure are also consistent with a healthy individual.   Fout-ta-ta-rou (B) is not a recognized term for a cardiac sound or rhythm.

Protodiastolic gallop rhythm (C) and atrial gallop rhythm (D) are terms that describe abnormal heart sounds and rhythms that are associated with heart failure and other cardiac conditions. IV physiological sound (E) is not a recognized term for a cardiac sound and is not typically heard in healthy individuals.


196. Estimation of community health level involved analysis of a report on diseases registered among the population of district under charge (reporting form 12). What index is

A. Common sickness rate

B. Index of pathological affection

C. Index of morbidity with temporary disability

D. Index of hospitalized morbidity

E. Index of basic non-epidemic morbidity

Answer: Common sickness rate

 Explanation

The common sickness rate is an index that measures the prevalence of illness in a population over a specified period of time. It is calculated by dividing the number of cases of illness by the total population and expressing the result as a percentage.

The common sickness rate is a useful indicator of the overall health status of a population, as it takes into account both acute and chronic conditions.   The index of pathological affection (B) is not a commonly used term in public health and is not a recognized indicator of community health.

The index of morbidity with temporary disability (C) is a measure of the prevalence of illness that results in temporary disability or loss of work time. The index of hospitalized morbidity (D) is a measure of the prevalence of illness that requires hospitalization.

The index of basic non-epidemic morbidity (E) is a measure of the prevalence of non-infectious diseases in a population.   In summary, the common sickness rate is the most appropriate index for measuring the prevalence of illness in a population based on the report on diseases registered among the population of district under charge (reporting form 12).


197. A 37 year old male patient was admitted to the resuscitation department because of attacks of tonoclonic spasms repeating every half an hour. Between the attacks the patient remains unconscious. AP is 120/90 mm Hg, Ps- 100 bpm. A day before the patient was at wedding and consumed alcohol. 5 years ago he had a closed craniocerebral trauma and brain contusion that later caused single convulsive attacks accompanied by loss of consciousness, but the patient didn’t undergo antiepileptic treatment. What drug should be injected for emergency aid?

A. Diazepam

B. Magnesium sulfate

C. Sodium oxybutyrate

D. Aminazine

E. Sodium thiopental


Answer:  Diazepam

 Explanation

The patient is experiencing recurrent tonic-clonic seizures with loss of consciousness, which is consistent with a diagnosis of status epilepticus. Status epilepticus is a medical emergency that requires immediate treatment to prevent brain damage and other complications.  

Diazepam is a benzodiazepine that is commonly used for the emergency treatment of status epilepticus. It works by enhancing the inhibitory action of GABA, a neurotransmitter that helps to regulate neuronal activity in the brain. Diazepam can be administered intravenously, intramuscularly, or rectally. It has a rapid onset of action and can quickly terminate seizures.  

Magnesium sulfate (B) is sometimes used as an adjunctive therapy for the treatment of seizures, particularly in cases of eclampsia and pre-eclampsia. It is not typically used as a first-line therapy for status epilepticus.   Sodium oxybutyrate (C) is a central nervous system depressant that is sometimes used to treat narcolepsy and other sleep disorders. It is not typically used for the emergency treatment of seizures.  

Aminazine (D) is an antipsychotic medication that is sometimes used to treat agitation and psychosis. It is not typically used for the treatment of seizures.   Sodium thiopental (E) is a short-acting barbiturate that is sometimes used as a general anesthetic or for the induction of therapeutic coma in cases of refractory status epilepticus. It is not typically used as a first-line therapy for the emergency treatment of seizures.


198. A 43 year old female patient was delivered to the hospital in grave condition. She suffers from Addison’s disease. The patient had been regularly taking prednisolone but a week before she stopped taking this drug. Objectively: sopor, skin and visible mucous membranes are pigmented, skin and muscle turgor is lowered. Heart sounds are muffled, rapid. AP- 60/40 mm Hg, heart rate – 96/min. In blood: Na120 millimole/l, K- 5,8 micromole/l. Development of this complication is primarily caused by the deficit of the following hormone:

A. Cortisol

B. Corticotropin (ACTH)

C. Adrenaline

D. Noradrenaline

E. Adrostendion


Answer: Cortisol

 Explanation

Addison’s disease is a condition characterized by the chronic underproduction of adrenal hormones, including cortisol and aldosterone. Cortisol is an important hormone that helps to regulate blood sugar levels, blood pressure, and immune function. In patients with Addison’s disease who are not receiving adequate hormone replacement therapy, the sudden withdrawal of cortisol can lead to a life-threatening condition known as an Addisonian crisis.  

The symptoms described in this case, including pigmented skin and mucous membranes, low skin and muscle turgor, muffled heart sounds, and low blood pressure, are all consistent with an Addisonian crisis. The rapid heart rate may be a compensatory response to the low blood pressure.   The deficit of cortisol is primarily responsible for the development of an Addisonian crisis. Cortisol helps to maintain blood pressure by regulating the body’s response to stress and by promoting the retention of sodium and water in the kidneys.

In the absence of cortisol, blood pressure can drop rapidly, leading to shock and other complications.   Corticotropin (ACTH) (B) is a hormone that stimulates the production of cortisol in the adrenal glands.

Adrenaline (C) and noradrenaline (D) are hormones produced by the adrenal medulla that help to regulate the body’s response to stress. Adrostendion (E) is a precursor hormone that is converted to testosterone and other sex hormones in the body. While these hormones may play a role in the development of some conditions, they are not primarily responsible for the development of an Addisonian crisis.


199. A 67 year old female patient complains about edemata of face and legs, pain in the lumbar area that is getting worse at moving; great weakness, sometimes nasal haemorrhages, rise of body temperature up to 38, 4oC. Objectively: painfulness of vertebral column and ribs on palpation. Laboratorial study revealed daily proteinuria of 4,2 g, ESR52 mm/h. What changes of laboratory indices are to be expected?

A. Whole protein of blood serum – 101 g/l

B. Leukocytes – 15,3 g/l

C. Haemoglobin – 165 g/l

D. Albumins – 65%

E. γ-globulins – 14%


Answer: Whole protein of blood serum – 101 g/l

 Explanation

The symptoms and laboratory findings described in this case are consistent with a diagnosis of multiple myeloma, a type of blood cancer that affects plasma cells in the bone marrow. Multiple myeloma can cause bone pain, weakness, and fatigue, as well as kidney damage that can lead to proteinuria.  

Proteinuria is a common finding in multiple myeloma, and the degree of proteinuria is often used as a marker of disease severity. In this case, the patient’s daily proteinuria of 4.2 g is indicative of significant kidney dysfunction. As a result, it is likely that the total protein level in the blood serum will be elevated, as the kidneys play an important role in regulating protein levels in the blood.

Therefore, option A, whole protein of blood serum – 101 g/l, is the correct answer.   Leukocytes (B), hemoglobin (C), albumins (D), and γ-globulins (E) are not specific laboratory findings for multiple myeloma and may be within normal limits or show different changes depending on the individual case. In general, multiple myeloma can cause anemia, leukopenia or leukocytosis, and alterations in blood protein levels, including decreased albumin and increased gamma globulins. However, the specific changes will depend on the individual patient and the extent of the disease.


200. A female patient consulted a doctor about gain in weight, chill, edemata, dry skin, sleepiness, problems with concentration. Objectively: the patient’s height is 165 cm, weight is 90 kg, gynoid body proportions, t o- 35, 8oC, ESR58/min, AP- 105/60 mm Hg. Heart sounds are weakened, bradycardia is present. Other internal organs have no changes. Thyroid gland is not palpable. Mammary glands ooze milk droplets. Hormonal study revealed rise of TSH and prolactin concentration, reduction of T4. What factor caused obesity?

A. Primary hypothyroidism

B. Secondary hypothyroidism

C. Prolactinoma

D. Hypopituitarism

E. Adiposogenital dystrophy


Answer:  Primary hypothyroidism

 Explanation

The symptoms and laboratory findings described in this case are consistent with a diagnosis of primary hypothyroidism. Hypothyroidism is a condition in which the thyroid gland does not produce enough thyroid hormone, which can lead to a wide range of symptoms, including weight gain, cold intolerance, dry skin, fatigue, and cognitive impairment.  

In this case, the patient’s symptoms of weight gain, chill, edema, dry skin, sleepiness, and problems with concentration are all consistent with hypothyroidism. The presence of bradycardia and weakened heart sounds may also be related to the decreased metabolic rate that can occur in hypothyroidism.  

The hormonal study revealed an increase in TSH and prolactin concentrations and a reduction of T4, which are all consistent with primary hypothyroidism. In primary hypothyroidism, the thyroid gland itself is unable to produce enough thyroid hormone, leading to an increase in TSH levels as the body tries to compensate for the low thyroid hormone levels.  

Prolactinoma (C) is a type of pituitary tumor that can cause an increase in prolactin levels and a range of symptoms, including galactorrhea (milk production), but it is not typically associated with weight gain and other symptoms of hypothyroidism.  

Secondary hypothyroidism (B) and hypopituitarism (D) are conditions in which the pituitary gland does not produce enough thyroid-stimulating hormone (TSH) or other pituitary hormones, leading to decreased thyroid hormone production. However, these conditions are less common than primary hypothyroidism.  

Adiposogenital dystrophy (E) is a rare condition characterized by obesity, delayed puberty, and other hormonal imbalances, but it is not typically associated with the specific hormonal abnormalities and symptoms described in this case.
Join the conversation
0% Complete